What is 4 2/3 + 1/6?

Answers

Answer 1
I hope this is the right equation. Hope this helps. <3
What Is 4 2/3 + 1/6?
Answer 2

Answer: 4 2/3 + 1/6 = 4 5/6

Step-by-step explanation:

4 2/3 + 1/6

Add whole numbers 4+0:   4

Combine fractions 2/3 + 1/6:   5/6

= 4 + 5/6 = 4 5/6


Related Questions

The junior class is selling candy bars for $1 each and Wildcat yard signs for $10 each.

Answers

Answer:

What the question???

Step-by-step explanation:

Woops did not mean to click add a answer

Solve the equation (-20)-2+9+18

Answers

Answer:

5

Step-by-step explanation:

The answer to this is 5

evaluate the expression when c=6 and d=8
d+4c​

Answers

Answer:

32

Step-by-step explanation:

d+4c

=8+4(6)

=8+24

=32

Answer:

32

Step-by-step explanation:

= d+4c

=8+4*6

=8+24

=32

 Nates architectural client said she wanted the worth of every room in your house increase by 2 feet and the length increase by 5 feet. The polynomial 2s^2-w-20 gives the area of any room in the house with w representing the rooms with. The width of the kitchen is 16 feet. What is the area of the kitchen?

Answers

Answer: 3(19)^2 - 19 - 20

3(361)-39

1083-39

1044

The area is 1044 ft.^2

The area is 1044

Step-by-step explanation:

Pleaseeeeeeeeeeeeeeeeeeee

Answers

Answer:

∠DAB = 68

Step-by-step explanation:

∠DAB we need to find this

∠DAB = ∠C (vertically opposite angles are equal)

∠DAB = 68

HELP ME AS FAST AS POSSIBLE! 95 POINTS!!!

Choose the equation below that is equivalent to 4(x-7) -2(x+1) = -10


A. -2(x - 7) + (x + 1) = -5

B. -2(x - 7) + (x + 1) = 5

C. 2(x - 7) + (x + 1) = -5

D. 2(x - 7) - (x + 1) = 5

Answers

Answer:

B

Step-by-step explanation:

4 (x - 7) - 2 (x + 1) = 10

x = 10

B. -2 (x - 7) + (x + 1) = 5

x = 10

your equation is equivalent to B!

Answer: -2(x-7) +(x +1)= 5

Step-by-step explanation:

Question 14 of 25
Simplify the expression and enter your answer below.
(171/7)7
Answer here
SUBMI

Answers

Answer:

171

Step-by-step explanation:

To simplify the expression:(171/7) × 7 =171

Ash makes essential oil using 5 drops of rose oil for every 1 drop of lavender oil.
How many drops of lavender oil would Ash need if they use 15 drops of rose oil?
CLEAR
CHECK
3
6
15
5
18

Answers

3 drops of lavender oil
15/5=3

Answer:

3

Step-by-step explanation:

its easy

Members of a baseball team raised $1781.25 to go to a tournament. They rented a bus for $1063.50 and budgeted $65.25 per player for meals. Write and solve an equation which can be used to determine x x, the number of players the team can bring to the tournament.

Answers

Answer:x=11

65.25x + 1063.50= 1781.25

To solve this equation, subtract the rental cost of the bus to get $717.75.

Now the equation should be 65.25x= 717.75

Divide both sides by 65.25, and you should get that x = 11. That’s your finally answer!

Step-by-step explanation:

Let f(x) = 12 over the quantity of 4 x + 2. Find f(−1). (1 point)

Answers

Answer:

-6

Step-by-step explanation:

Hope I understand your "f(x) = 12 over the quantity of 4 x + 2" correctly

f(x) = 12 / 4x +2

Now so they want a f(-1), as u can see they substituted the x for - 1, therefor do that the same to the right side

f(-1) = 12/ 4(-1) + 2

f(-1) = 12 / - 4+2

= 12/ - 2

= - 6

I know the answer, I'm just confused on what to put for explore, plan, solve, and examine!

Joseph earned $53 the first week he worked at Jim’s Candy Store and $62 the next week. How much did Joseph earn his first two weeks?
Explore:
Plan:
Solve:
Examine:​

Answers

Answer:

Explore - ??? (What are you suppose to put for that (Add comment))

Plan - First week - 53 Second week - 62

Solve - 53 + 62 = 115

Examine - (Use a graph to see the rise?)

Step-by-step explanation:

Can anybody help find the line of reflection??

Answers

I'll do part (a) to get you started. Part (b) will follow the same sort of steps.

In part (a), we have these two points

A = (-3,2)B = (3,8)

You can call the points any letters you want. Let's find the slope of line AB

m = (y2-y1)/(x2-x1)

m = (8-2)/(3-(-3))

m = (8-2)/(3+3)

m = 6/6

m = 1

The slope of line AB is 1.

The mirror line will have the negative reciprocal of this slope, so the slope of the mirror line is -1. In other words, the mirror line slope is perpendicular to line AB. We'll use this perpendicular slope later.

-------------------------------------------

Next, we find the midpoint of segment AB

Average the x coordinates to get (x1+x2)/2 = (-3+3)/2 = 0/2 = 0

Repeat for the y coordinates:  (y1+y2)/2 = (2+8)/2 = 10/2 = 5

The midpoint of AB is (0,5)

-------------------------------------------

The mirror line will have two defining traits

The slope of the mirror line is m = -1The mirror line goes through the midpoint (x,y) = (0,5)

So,

y = mx+b

y = -1x+b ... plugging in that perpendicular slope

5 = -1*0+b .... plugging in (x,y) = (0,5)

5 = 0+b

b = 5

The equation of the mirror line is y = -x+5 as shown in the diagram below. The mirror line is shown in green. Point D isn't important as it's just used to help set up the 90 degree angle.

This wraps up part (a) and I'll let you handle part (b). If you're stuck, then feel free to ask.

Rewrite the equation in slope-intercept form. Then identify the slope and the y intercept of the line
4x + y = 1

Answers

4x = 1 - y ____eqn I

Answer:

see explanation

Step-by-step explanation:

The equation of a line in slope- intercept form is

y = mx + c ( m is the slope and c the y- intercept )

Given

4x + y = 1 ( subtract 4x from both sides )

y = - 4x + 1 ← in slope- intercept form

with slope m = - 4 and y- intercept c = 1

SOMEBODY PLEASE ANSWER THIS IVE BEEN TRYING FOR 2 DAYS NOW I NEED HELP. The value of a car is depreciating by 12% per year.
Write a rule expressing the value of the car after x years if it is currently worth $15,000.
What will the car be the worth in 10 years ? What was the car worth 5 years ago ?

Answers

If it is depreciating by 12% per year it forms geometric sequence with a common ratio of 100% - 12% = 88% = 0.88
The value in x years = 15000 x 0.88^x

Value in 10 years = 15000 x 0.88^10 = $4178 (to nearest dollar)

Value 5 years ago = 15000 x 0.88^-5 = $28423 (to nearest dollar)

5x -4= 4x please help me

Answers

[tex]5x-4=4x\\\\5x-4x=4\\\\\huge\boxed{x=4}[/tex]

Answer:

4=x

Step-by-step explanation:

5x-4=4x

-4=-x

4=x

A polynomial f (x) has the given zeros of 8, –1, and –3.

Part A: Using the Factor Theorem, determine the polynomial f (x) in expanded form. Show all necessary calculations. (3 points)

Part B: Divide the polynomial f (x) by (x2 – x – 2) to create a rational function g(x) in simplest factored form. Determine g(x) and find its slant asymptote. (4 points)

Part C: List all locations and types of discontinuities of the function g(x). (3 points)

Answers

The answer is "[tex]\bold{x^3-4x^2-29x-24, y=x-3,\ and\ x=2}[/tex]" and following are the complete solution to the given parts:

For point A)[tex]\to \bold{f(x)=(x-8)(x+1)(x+3)=(x^2-7x-8)(x-8)=(x^3-4x^2-29x-24)}[/tex]For point B)

[tex]\to \bold{x^2-x-2=(x-2)(x+1)}\\\\\to \bold{g(x)=\frac{(x-8)(x+1)(x+3)}{(x-2)(x+1)}}[/tex]

When  [tex]x \neq -1[/tex] then  

[tex]\to \bold{g(x)=\frac{(x-8)(x+3)}{x-2}}\\\\\to \bold{\lim_{n \to \infty} \frac{g(x)}{x}}[/tex]

So  

For point C)

It is discontinuity when a [tex]\bold{x=2}[/tex] that is a vertical asymptote.

Learn more:

brainly.com/question/24520259

Answer:

Part A:

x^3 - 4x^2 - 29x -24

(x + 1)(x - 8)(x + 3)

Part B.

(x + 1)(x - 8)(x + 3) / x^2 - x - 2= (x + 3)(x - 8) / x - 2

g(x)=  (x + 3)(x - 8) / x - 2= y= x - 3

Part C.

x=2.

Step-by-step explanation:

See images for more information, it wouldn't let me post it.

Please just help with these questions!

Answers

5/7=        -----------

           0.714285

Convert the fraction to a decimal by dividing the numerator by the denominator.

6/10= 0.6

Convert the fraction to a decimal by dividing the numerator by the denominator.

16/20= 0.8

Convert the fraction to a decimal by dividing the numerator by the denominator.

9/10= 0.9

Convert the fraction to a decimal by dividing the numerator by the denominator.  

i need help plzzzzzzzz

Answers

Answer:

Step-by-step explanation:

The expanded form of 79 is 70+ 9. To apply the distributive property you rewrite the problem 3(70 + 9) which is the same as (3x70) + (3x9). Next do the multiplication 3x70= 210 + 3x9= 27

Next add the sums: 210 + 27 = 237.

It rained 12 days out of 30 days. Which
ratio compares the number of rainy days to
the number of dry days?

Answers

The answer to your question is 4:10

Someone Please help me on this question!

Answers

Answer:

A. [tex]g(x) = \sqrt[3]{x} + 3[/tex]

a=b-c
How do I write it as c=?
and the equation

Answers

Answer:

The Quadratic Formula uses the "a", "b", and "c" from "ax2 + bx + c", where "a", "b", and "c" are just numbers; they are the "numerical coefficients" of the quadratic equation they've given you to solve.

Step-by-step explanation:

Here, hope it helps :)

find 2^5 root 4 root (2^3)^4​

Answers

Step-by-step explanation:

[tex]2^5\cdot\sqrt4\cdot(2^3)^4[/tex]

[tex]=2^5\cdot4^\frac12\cdot(2^3)^4[/tex]

[tex]=2^5\cdot(2^2)^\frac12\cdot(2^3)^4[/tex]

[tex]=2^5\cdot2^{2\cdot\frac12}\cdot2^{3\cdot4}[/tex]

[tex]=2^5\cdot2^1\cdot2^{12}[/tex]

[tex]=2^{5+1+12}[/tex]

[tex]=2^{18}[/tex]

The graph of y = f(x) is graphed below. What is the end behavior of f(x) ?

Answers

Answer:

The first option, as x approaches infinity, y approaches infinity and as  x approaches negative infinity, y approaches negative infinity

Step-by-step explanation:

find the distance between the points (3, 7) and (2, 6)​

Answers

Answer:

1.414214

Step-by-step explanation:

I think this is right lol.

18)
Christopher's cell phone company
charges a monthly'rate of $56 plus
$0.95 per gigabyte of data used,
and $0.08 per text message sent.
Christopher sent 55 text messages
messages in April and received a bill
for $68.00. How much data did he
use?

Answers

Answer:

8 GB

Step-by-step explanation:

Cost = sum of fees 1, 2, and 3

C = 56 + .95g +  .08t

so

68 = 56 + .95g + .08(55)

68 = 56 + .95g + .08(55)

68 = 56 + .95g + 4.4

68 - 56 - 4.4 = .95g

7.6 = .95 g

7.6/.95 = g

8 = g

plsss helpppp will mark brainliest!!

Answers

Answer:

D. 2¹⁵/4¹⁰

Step-by-step explanation:

(2³/4²)⁵ = 1/64

A. 2⁸/4⁷ = 1/128

B. 2⁵/4⁴ = 1/16

C. 2⁶/4⁶= 1/128

D. 2¹⁵/4¹⁰= 1/64

SO (2³/4²)⁵ = 2¹⁵/4¹⁰

Please help, I can’t do this

Answers

Answer:

Choice 3. -4x-3<13

Step-by-step explanation:

First, you will use process of elimination and cross out any obvious wrong answers. You can see that the symbol would not be flipped yet so the "<" would be pointing  to less than. That eliminates choice 1 and 4. Now, You can observe the 3rd step and see that -4x<16, to get that, you would need a "-4x" in the equation. Number 2 is 2x so that can't be right. Therefore, the answer is choice 3. This simple process of elimination and looking for clues will help you greatly in multiple choice problems instead of solving it the hard way.

Answer: C

Step-by-step explanation:

[tex]-x-3 < 13+3x\\\\\Longleftrightarrow\ -x-3x-3 < 13\\\\this \ step \ is \ missing\ -4x-3 < 13\\\\\Longleftrightarrow\ -4x < 13 + 3\\\\\Longleftrightarrow\ -4x < 16\\\\\Longleftrightarrow\ 4x > -16\\\\\Longleftrightarrow\ x > -4\\\\Answer\ C[/tex]

ers
17 Which number is greater, -7 or 6? Which number has the greater
absolute value, -7 or 6? Explain your thinking. Use comparison
symbols and absolute value symbols when you write your answer.
Solution

Answers

Answer:

17

Step-by-step explanation:

17-38 8 im like hey whats up hello

Find the LCM of 13 and 52.

LCM =
.

Answers

Answer:

Step-by-step explanation:

13 , 52

Here 13 is a prime number. So, check if 52 is  a multiple of 13.

Yes, 13*4 = 52

LCM = 52

Answer:

52

Step-by-step explanation:

L.C.M of 13 and 52 is,

13  | 13 , 52

4  |  1 , 4

     | 1 ,  1

L.C.M = 13 x 4 = 52

Decide!!!!!!!!!!!!!!

Answers

Answer:

547

Step-by-step explanation:

The minute hand hits the 0 minute once each hour. This results each hour to become 61 minutes.

Number of hours from Monday 00:01 to Friday 07:30 is 4*24 + 7.5 = 103.5

Extra time considering defective clock (full hours):

103 minutes = 1 hr and 43 min

Alarm to be set at:

07:30 - 1:43 = 6:00 - 0:13 = 05:47

The answer is:

5*100 + 47 = 547
Other Questions
[tex](x^2+3)^{-1/3} -2/3x^2(x^2+3)^{-4/3}[/tex]Factor the expression completely. Question 14 Multiple Choice Worth 1 points)(01.01 LC)Carlos wants to participate in a physical activity that will best improve his heart function. Which type of activity should hechoose? help asap please 7 grade math If placed on a number line, 502 would fall between the whole numbersWhats the answer? CoomementsThe formula for the volume of a cylinder is V = xrh. The cylinder to the right has an exact volume of 720x cubicmeters. Find its height..6 mThe height of the cylinder is(Simplify your answer.) what is socilization True or false: it would take 100 years on a space shuttle to reach the closest galaxy The number of viewers of a television station in 2003 was 640000.(a) In 2004, the station had 716800 viewers. Calculate the percentage increase in the number of viewers from 2003 to 2004.Each year, the television station invites it's viewers to vote for their favourite actors and actresses.(b) In 2005, the number of viewers was 896000. Of those viewers, 108000 voted.(i) Calculate the percentage of viewers who voted in 2005, giving your answer correct to the nearest whole number.(ii) The number of viewers who voted in 2005 was 20% more than that in 2004. Calculate the number of viewers who voted in 2004.______________________________State your answers with the method of how you got your answers. 3what is not example of an Internal Stakeholder?-OOwnerCustomersManagerEmployeesNone of the Above What might have been some of the advantages and disadvantages of taking a long voyage in a caravel? Do the calculations and round the answers to the correct place (using the rules for calculating significant digits): 4.8754 x 3.2 8. Is the following statement true or false?It is not important to take good notes when reviewing research sources so that you can stay organized and ind a source later if you need tofalsetrue Which of the following is true in a command economy? A. Consumers have many choices. B. The government makes no economic decisions. C. The government decides which goods are produced. D. Business owners decide which goods to produce. 3. The number of companies producing computers increases by 20%. What would happen tothe supply of computers? Analiza la siguiente estrofa: Uno no escoge el pas donde nace; pero ama el pas donde ha nacido". 2 tacos cost $6. What is the cost for 6 tacos? the process by which charged plasma particlesrevert back to a gas Study the pattern of numbers given below and write the rule that describes the pattern 1,3,9,27,81 b/4 = 2 solving one-step equations ii Ifig of fully saturated haemoglobin iscombined with 1.3 cmof oxygen, how muchoxygen will 1g of haemoglobin in the capillariesin the lungs be combined with?